Defintion of e as a limit

  • Thread starter blerg
  • Start date
  • Tags
    Limit
In summary: Since e is a constant, this shows that x_n is bounded below by e, which means that x_n is decreasing and bounded below.In summary, we have shown that x_n = \left(1 + \frac{1}{n}\right)^{n+1} is decreasing and bounded below.
  • #1
blerg
66
0

Homework Statement


Show that [tex] x_n = \left(1 + \frac{1}{n}\right)^{n+1}[/tex] is decreasing and bounded below


Homework Equations





The Attempt at a Solution


I have tried all sorts of things and none some to be working. I tried expanding it using the binomial theorem, but there are more terms in [tex] x_{n+1} [/tex] than [tex] x_n [/tex] so i didn't see an easy way to compare them. I tried looking at [tex] \frac{x_{n+1}}{x_n} [/tex] and [tex] x_nx_{n+1} [/tex] with nothings giving me and progress. Any suggestions?
 
Physics news on Phys.org
  • #2


Dear student,

To show that x_n is decreasing, we can take the derivative of x_n and show that it is always negative. Let's start by rewriting x_n as:

x_n = \left(1 + \frac{1}{n}\right)^{n+1} = \left(\frac{n+1}{n}\right)^{n+1}

Now, taking the natural logarithm of both sides, we get:

\ln(x_n) = \ln\left[\left(\frac{n+1}{n}\right)^{n+1}\right] = (n+1)\ln\left(\frac{n+1}{n}\right)

Next, we can take the derivative of both sides with respect to n:

\frac{d}{dn}\ln(x_n) = \frac{d}{dn}\left[(n+1)\ln\left(\frac{n+1}{n}\right)\right]

Using the product rule and the chain rule, we get:

\frac{1}{x_n}\cdot\frac{dx_n}{dn} = \ln\left(\frac{n+1}{n}\right) + (n+1)\cdot\frac{1}{\frac{n+1}{n}}\cdot\left(\frac{n}{n+1}\right)^2

Simplifying this expression, we get:

\frac{dx_n}{dn} = -\frac{n+1}{n^2}\left[\ln\left(\frac{n+1}{n}\right) - \frac{n}{n+1}\right]

Now, we can see that the expression in the square brackets is always negative, since n > 0. Therefore, the derivative of x_n is always negative, which means that x_n is decreasing.

To show that x_n is bounded below, we can use the fact that e = \lim_{n\to\infty}\left(1 + \frac{1}{n}\right)^n. This means that for any n > 0, we have:

\left(1 + \frac{1}{n}\right)^n < e

Multiplying both sides by \left(1 + \frac{1}{n}\right), we get:

\left(1 + \frac{1}{n}\right)^{n+1} < e\left(1 + \frac{1}{n}\right
 

1. What is the definition of e as a limit?

The definition of e as a limit is the mathematical constant e, which is approximately equal to 2.71828, can be defined as the limit of (1+1/n)^n as n approaches infinity.

2. Why is e considered an important number in mathematics?

E is considered an important number in mathematics because it appears in many important formulas and equations, such as compound interest, exponential growth and decay, and the normal distribution. It also has many interesting properties and connections to other mathematical concepts.

3. How is e related to the natural logarithm?

E is the base of the natural logarithm, ln. This means that e to the power of a number will give the natural logarithm of that number, and the natural logarithm of e is equal to 1.

4. What is the significance of e being an irrational number?

E is an irrational number, which means it cannot be expressed as a finite decimal or fraction. This makes it a unique and important number in mathematics, as it cannot be represented by a simple ratio of two integers.

5. How is e used in calculus?

E is used in calculus to model natural growth and decay, as well as to calculate derivatives and integrals of exponential functions. It is also used in the definition of the natural logarithm, which is an important concept in calculus.

Similar threads

  • Calculus and Beyond Homework Help
Replies
5
Views
987
  • Calculus and Beyond Homework Help
Replies
12
Views
1K
  • Calculus and Beyond Homework Help
Replies
3
Views
1K
  • Calculus and Beyond Homework Help
Replies
12
Views
1K
  • Calculus and Beyond Homework Help
Replies
2
Views
833
  • Calculus and Beyond Homework Help
Replies
6
Views
475
Replies
12
Views
881
  • Calculus and Beyond Homework Help
Replies
7
Views
1K
  • Calculus and Beyond Homework Help
Replies
1
Views
1K
Replies
11
Views
1K
Back
Top